Difference between revisions of "2003 AMC 12A Problems/Problem 9"

(solution added)
(added see also)
Line 6: Line 6:
 
== Solution ==
 
== Solution ==
 
If <math>(2,3)</math> is in <math>S</math>, then <math>(3,2)</math> is also, and quickly we see that every point of the form <math>(\pm 2, \pm 3)</math> or <math>(\pm 3, \pm 2)</math> must be in <math>S</math>. Now note that these <math>8</math> points satisfy all of the symmetry conditions. Thus the answer is <math>D</math>.
 
If <math>(2,3)</math> is in <math>S</math>, then <math>(3,2)</math> is also, and quickly we see that every point of the form <math>(\pm 2, \pm 3)</math> or <math>(\pm 3, \pm 2)</math> must be in <math>S</math>. Now note that these <math>8</math> points satisfy all of the symmetry conditions. Thus the answer is <math>D</math>.
 +
 +
== See Also ==
 +
*[[2003 AMC 12A Problems]]
 +
*[[2003 AMC 12A/Problem 8|Previous Problem]]
 +
*[[2003 AMC 12A/Problem 10|Next Problem]]
 +
 +
[[Category:Introductory Algebra Problems]]

Revision as of 11:46, 16 November 2008

Problem

A set $S$ of points in the $xy$-plane is symmetric about the orgin, both coordinate axes, and the line $y=x$. If $(2,3)$ is in $S$, what is the smallest number of points in $S$?

$\mathrm{(A) \ } 1\qquad \mathrm{(B) \ } 2\qquad \mathrm{(C) \ } 4\qquad \mathrm{(D) \ } 8\qquad \mathrm{(E) \ } 16$

Solution

If $(2,3)$ is in $S$, then $(3,2)$ is also, and quickly we see that every point of the form $(\pm 2, \pm 3)$ or $(\pm 3, \pm 2)$ must be in $S$. Now note that these $8$ points satisfy all of the symmetry conditions. Thus the answer is $D$.

See Also